Question

In: Advanced Math

Prove that a subspace of R is compact if and only if it is closed and bounded.

Prove that a subspace of R is compact if and only if it is closed and bounded.

Solutions

Expert Solution

An unbounded subset of R has an open cover consisting of all bounded, open
intervals. This has no finite subcover, since the union of a finite set of bounded
intervals is bounded.
Similarly, if K is not closed then it has a boundary point x ∉ K. But then the collec-
tion of sets R\ [x −ε,x +ε] with ε > 0 is an open cover with no finite subcover.
We have shown that a set which is either unbounded or not closed is not compact.
It remains to prove that a set K ⊂ R which is both bounded and closed is compact.
First, we show that a closed and bounded interval [a,b] is compact.
Let O be an open cover of [a,b].
Define:
s = supC where ,
C =. {c ∈ [a,b]: O has a finite subcover for [a,c] }
It is clear that a ∈C, since O is a cover for [a,b]. So C is not empty, and s ∈ [a,b].
Pick some U ∈ O with s ∈U, and let ε > 0 so that [s −ε,s +ε] ⊆U.
Pick some c ∈ C with c > s −ε (it exists by the definition of s). Then there is a finite
subcover Q of O for [a,c]. But then Q∪ {U}
is a finite subcover of O for [a,c]
where,
Cl = min(s + ε,b). Thus cl ∈ C. If s < b then this contradicts the definition of s,
therefore s = b, and b = cl ∈C, in other words there is a finite subcover for [a,b].

Finally, let K ⊂ R be closed and bounded, and let O be an open cover of K. Then
U = R \K is open, and O ∪ {U} is an open cover of [a,b], where K ⊆ [a,b] (which
can be so arranged because K is bounded). Since [a,b] is compact, it has a finite
subcover Q. Now Q\{U} ⊆ O is a finite cover of K, and the proof is complete.


Related Solutions

1. Prove that any compact subset ot a metric space is closed and bounded. 2. Prove...
1. Prove that any compact subset ot a metric space is closed and bounded. 2. Prove that a closed subset of a compact set is a compact set.
Prove Corollary 4.22: A set of real numbers E is closed and bounded if and only...
Prove Corollary 4.22: A set of real numbers E is closed and bounded if and only if every infinite subset of E has a point of accumulation that belongs to E. Use Theorem 4.21: [Bolzano-Weierstrass Property] A set of real numbers is closed and bounded if and only if every sequence of points chosen from the set has a subsequence that converges to a point that belongs to E. Must use Theorem 4.21 to prove Corollary 4.22 and there should...
a) Suppose f:R → R is differentiable on R. Prove that if f ' is bounded...
a) Suppose f:R → R is differentiable on R. Prove that if f ' is bounded on R then f is uniformly continuous on R. b) Show that g(x) = (sin(x4))/(1 + x2) is uniformly continuous on R. c) Show that the derivative g'(x) is not bounded on R.
prove that a compact set is closed using the Heine - Borel theorem
prove that a compact set is closed using the Heine - Borel theorem
a. Prove that y=sin(x) is a subspace of R^2 b. Prove that a set of 2x2...
a. Prove that y=sin(x) is a subspace of R^2 b. Prove that a set of 2x2 non invertible matrices a subspace of all 2x2 matrices
Prove that {f(x) ∈ F(R, R) : f(0) = 0} is a subspace of F(R, R)....
Prove that {f(x) ∈ F(R, R) : f(0) = 0} is a subspace of F(R, R). Explain why {f(x) : f(0) = 1} is not.
Let S ⊆ R be a nonempty compact set and p ∈ R. Prove that there...
Let S ⊆ R be a nonempty compact set and p ∈ R. Prove that there exists a point x_0 ∈ S which is “closest” to p. That is, prove that there exists x0 ∈ S such that |x_0 − p| is minimal.
Let A be a bounded linear operator on Hilbert space. Show that if R(A) is closed,...
Let A be a bounded linear operator on Hilbert space. Show that if R(A) is closed, so is R(A*)
prove that a ring R is a field if and only if (R-{0}, .) is an...
prove that a ring R is a field if and only if (R-{0}, .) is an abelian group
Compact and analysis conception 1. Are all closed interval compact? for example [0,1]. are they closed...
Compact and analysis conception 1. Are all closed interval compact? for example [0,1]. are they closed and bounded? 2. If i can find the Maximum and Minimum, does that mean the set is closed and bounded?
ADVERTISEMENT
ADVERTISEMENT
ADVERTISEMENT